You are on page 1of 3

ELECTRIC CHARGES AND FIELDS 1

Online Chapter Tests


1. ELECTRIC CHARGES AND FIELDS

Time : 3 Hrs. MM : 45
General Instructions
1. This test consists of 28 questions, grouped according to their type, carrying a total of 45 marks.
2. Marks related to each question are mentioned clearly.
3. Time allotted to complete this test is 3 hrs.
4. It is advisable that you should attempt this test in a single slot of time and without the help of books or teachers.
5. To assess your chapterwise understanding, once you complete this test compare your answers/solutions with our
solutions given with each test and score yourself.

I. Multiple Choice Questions [1 Mark each]

1. Two charges, each equal to q are kept at x = − a and x = a on the X-axis. A particle of mass m and charge
q 1 = q /2 is placed at the origin. If charge q 1 is given a small displacement ( y < < a) along the Y-axis, the net
force acting on the particle is proportional to
(a) y (b) − y (c) 1/ y (d) −1/ y
ρ0
2. A solid sphere of radius R 1 and volume charge density ρ = is enclosed by a hollow sphere of radius R 2
r
with negative surface charge density σ, such that the total charge in the system is zero. ρ0 is a positive
constant and r is the distance from the centre of the sphere. The ratio R 2 / R 1 is
(a) σ/ρ 0 (b) 2σ /ρ 0 (c) ρ0 /2σ (d) ρ0 /σ
3. Charge on a body which carries 400 excess electron is
(a) − 3. 2 × 10 −18 C (b) − 6.4 × 10 −17 C (c) 3. 2 × 10 −19 C (d) 6.4 × 10 −18 C
4. Identify the correct diagram representing electrostatic field lines from the following diagrams.
F
+ –

(a) + – + (b) + q1 q2 – (c) (d) + –


+
(q1<q2)

5. The torque acting on the dipole formed due to two charges of ±1000 µC separated by 2 mm and held at an
angle of 30° with a uniform electric field of 15 × 10 4 NC−1, is
(a) 1.5 N-m (b) 0.15 N-m (c) 0.015 N-m (d) 15 N-m
II. True or False [1 Mark each]

6. A ring of radius r carries a charge Q uniformly distributed over its length. A charge q 1 is placed its centre will
experience no force.

Note: This chapter test is exclusively available at examwitharihant.com. All the rights related to it belong to examwitharihant.com.
2 Online Chapter Test

7. Two particle of equal mass m and charge q are placed at a distance of 25 cm. They do not experience any
force. The ratio of q /m is equal to 4 π ε G.
0

8. If a linear isotropic dielectric is placed in an electric field of E, then the polarisation P is directly
proportional to E.
9. The linear charge density of an infinite line charge producing a f ield of 9 × 10 4 N/C at a distance of 0.02 m
is 10 −9 C/m.
10. Electric flux through a cube of side 1 cm, which encloses an electric dipole is 0.

III. Fill in the Blanks [1 Mark each]

11. Direction of electric field due to electric dipole, depends on the angle between the ……… and ……… .
12. The difference of potential energy of an electric dipole in an uniform external electric field
U unstable − U stable = .......... .
13. SI unit of electric field intensity is …… and its a ……… quantity.
14. Electrostatic forces are ……… forces.
15. ……… is the dimensional formula of electric flux.

IV. Matching Type [2 Marks each]

16. Match the field lines given in Column I with the charge configuration due to which field lines exist in
Column II and mark the correct option from the codes given below.
Column I Column II

A. P. A single negative point charge


+ –

B. Q. A pair of equal negative point charges

+ +
N

C. R. Equal positive point charges

D. S. A pair of equal and opposite charges


– –
N

Codes
A B C D A B C D
(a) S R P Q (b) P R S Q
(c) S R P Q (d) Q P S R
17. A charge q 1 = − 4 µC is at the origin and a charge q 2 = − 2 µC is on the X-axis at x = 1 m. A net force is acting
on charge q3 = + 1 µC located at x = 0 .75 m. Match the Column I with Column II and mark the correct option
from the codes given below.
q1 q3 q2
– + –
0.75 m 1m
ELECTRIC CHARGES AND FIELDS 3

Column I Column II

A. Net force acting on q 3 P. 0.064 N, along X-axis

B. Force acting on q 3 due to q1 Q. 0. 288 N, along +ve X-axis.

C. Force acting on q 3 due to q 2 R. 0.064 N

D. Magnitude of force on q 3 due to q1 S. 0. 224 N, along + X-axis

Codes
A B C D A B C D
(a) S R Q P (b) Q P R S
(c) S P Q R (d) P Q R S
Very Short Answer Type Questions [1 Mark each]

18. Will the electric potential be zero at a point, if electric field intensity is zero at that point?
19. Tyres of the aircrafts are made slightly conducting. Justify this statement.
20. What will be the number of electric lines of force that radiate outwards from 5 C of charge in vacuum?
21. Will an electron and a proton kept in the same electric field experience same force and have same acceleration?
22. A charge Q is divided into two parts of q and Q – q. If the repulsion between them is maximum, when they
are kept at a distance r. Calculate the value of q.

Short Answer Type I Question [2 Marks each]

23. Find the electric field on axial as well as on equitorial line at a distance 10 cm from the mid-point of the
dipole, where two charges +10 µC and − 10 µC are 10 mm apart.

Short Answer Type II Questions [3 Marks each]

24. Five point charges, each of value +q are placed on five vertices of a regular hexagon to side L. What is the
magnitude of the force on a point charge of value − q placed at the centre of the hexagon?
25. A particle of charge q and mass m moves rectilinearly under the action of an electric field E = A − B x ,
where B is a positive constant and x is a distance from the point, where the particle was initially to rest.
Calculate (i) distance travelled by the particle till it comes to rest. (ii) acceleration at the moment.
26. The electrostatic force between two charges is 200 N. If someone increases 10% charge on one of the
charges and decreases 10% charge on the other charge while separation between them remains unchanged.
What is the new electrostatic force between them?

Long Answer Type Questions [5 Marks each]

27. (i) Name the physical quantity which represents the total number of electric lines of force passing
normally through that area and write its SI unit.
(ii) Three charges +a, +a and − 2a are placed at the vertices of an equilateral triangle. What is the dipole
moment of the system?
(iii) The electric field in a certain region of space is ( 5i$ + 4 $j − 4 k$ ) × 105 (N/C). Calculate electric flux due to
this field over an area of (2$i − $j) × 10 −2 m 2.
28. (i) A point charge q lies inside a sphere of radius r. If the radius of the sphere is doubled, how will the
electric flux be changed?
(ii) Two point charges q 1 and q 2 are 3 m apart and their combined charge is 20µC. If one repels the other
with a force of 0.075 N, what are the two charges?
(iii) What is the ratio of electric fields on the axis and at equator of an electric dipole?

You might also like